0 Daumen
248 Aufrufe

Aufgabe:

Berechne den Konvergenzradius folgender Potenzreihe:

\(\sum\limits_{n=1}^\infty \left(1+\frac{1}{8}+...+\frac{1}{n^3}\right)\cdot x^n \)

Wie könnte ich hier vorgehen.

Avatar von

\(n=0\) ist hier nicht möglich, da sonst durch Null geteilt wird. Habe es entsprechend angepasst.

2 Antworten

0 Daumen
 
Beste Antwort

Aloha :)

Wir schreiben zunächst die Funktionsgleichung etwas übersichtlicher:$$f(x)=\sum\limits_{n=1}^\infty\left(1+\frac{1}{8}+\cdots+\frac{1}{n^3}\right)\cdot x^n=\sum\limits_{n=1}^\infty\left(\sum\limits_{k=1}^n\frac{1}{k^3}\right)\cdot x^n\quad\implies$$$$f(x)=\sum\limits_{n=1}^\infty a_nx^n\quad;\quad a_n\coloneqq\sum\limits_{k=1}^n\frac{1}{k^3}$$und überlegen uns, ob die Reihe für \(a_n\) konvergiert oder nicht. Normalerweise schätzt man eine solche Summe wie folgt ab:$$a_n=\sum\limits_{k=1}^n\frac{1}{k^3}\le\sum\limits_{k=1}^n\frac{1}{k^2}<2$$weil man (z.B. aus der Vorlesung oder aus den Übungen) weiß, dass \(\sum\limits_{k=1}^n\frac{1}{k^2}\) konvergiert. Falls du das (noch) nicht weißt, kannst du das wie folgt zeigen:

$$\sum\limits_{k=1}^n\frac{1}{k^2}=1+\sum\limits_{k=2}^n\frac{1}{k\cdot k}<1+\sum\limits_{k=2}^n\frac{1}{k(k-1)}=1+\sum\limits_{k=2}^n\left(\frac{1}{k-1}-\frac{1}{k}\right)$$$$\phantom{\sum\limits_{k=1}^n\frac{1}{k^2}}=1+\sum\limits_{k=2}^n\frac{1}{k-1}-\sum\limits_{k=2}^n\frac{1}{k}=1+\sum\limits_{k=1}^{n-1}\frac{1}{k}-\sum\limits_{k=2}^n\frac{1}{k}$$$$\phantom{\sum\limits_{k=1}^n\frac{1}{k^2}}=1+\left(\frac{1}{1}+\sum\limits_{k=2}^{n-1}\frac{1}{k}\right)-\left(\sum\limits_{k=2}^{n-1}\frac{1}{k}+\frac{1}{n}\right)=1+1-\frac{1}{n}=2-\frac{1}{n}\to2$$

Damit ist nun klar, dass \(1\le a_n<2\) für alle \(a_n\) gilt und wir können den Konvergenzradius der Funktion \(f(x)\) als Grenzwert \(r\) des folgenden Quotienten bestimmen:

$$\left|\frac{a_n}{a_{n+1}}\right|=\frac{\sum\limits_{k=1}^n\frac{1}{k^3}}{\sum\limits_{k=1}^{n+1}\frac{1}{k^3}}=\frac{\sum\limits_{k=1}^n\frac{1}{k^3}}{\sum\limits_{k=1}^{n}\frac{1}{k^3}+\frac{1}{(n+1)^3}}=\frac{\sum\limits_{k=1}^n\frac{1}{k^3}+\frac{1}{(n+1)^3}-\frac{1}{(n+1)^3}}{\sum\limits_{k=1}^{n}\frac{1}{k^3}+\frac{1}{(n+1)^3}}$$$$\phantom{\left|\frac{a_n}{a_{n+1}}\right|}=1-\frac{\frac{1}{(n+1)^3}}{\sum\limits_{k=1}^{n}\frac{1}{k^3}+\frac{1}{(n+1)^3}}=1-\frac{1}{(n+1)^3\left(\sum\limits_{k=1}^{n}\frac{1}{k^3}+\frac{1}{(n+1)^3}\right)}$$$$\phantom{\left|\frac{a_n}{a_{n+1}}\right|}=1-\frac{1}{1+(n+1)^3\sum\limits_{k=1}^{n}\frac{1}{k^3}}=1-\frac{1}{1+(n+1)^3\cdot a_n}$$Wegen \(1\le a_n<2\) konvergiert der Nenner gegen unendlich, sodass gilt:$$r=\lim\limits_{n\to\infty}\left|\frac{a_n}{a_{n+1}}\right|=1$$

Avatar von 149 k 🚀
0 Daumen

Betrachte die Summe im Klammerausdruck, also \(1+\frac{1}{8}+...+\frac{1}{n^3}=\sum\limits_{k=1}^n\frac{1}{k^3}\). Welches Konvergenzkriterium kannst du heranziehen?

Avatar von 15 k

Ein anderes Problem?

Stell deine Frage

Ähnliche Fragen

1 Antwort
Gefragt 9 Dez 2015 von Gast
1 Antwort
0 Antworten

Willkommen bei der Mathelounge! Stell deine Frage einfach und kostenlos

x
Made by a lovely community